Menu Close

x-2-1-x-4-1-dx-




Question Number 86491 by john santu last updated on 29/Mar/20
∫  ((x^2 +1)/(x^4 +1)) dx ?
$$\int\:\:\frac{\mathrm{x}^{\mathrm{2}} +\mathrm{1}}{\mathrm{x}^{\mathrm{4}} +\mathrm{1}}\:\mathrm{dx}\:?\: \\ $$
Commented by john santu last updated on 29/Mar/20
dear prof mr mjs. what the super  easy method ?
$$\mathrm{dear}\:\mathrm{prof}\:\mathrm{mr}\:\mathrm{mjs}.\:\mathrm{what}\:\mathrm{the}\:\mathrm{super} \\ $$$$\mathrm{easy}\:\mathrm{method}\:? \\ $$
Answered by som(math1967) last updated on 29/Mar/20
∫(((x^2 +1)/x^2 )/((x^4 +1)/x^2 ))dx  ∫((1+(1/x^2 ))/(x^2 +(1/x^2 )))dx  ∫((d(x−(1/x)))/((x−(1/x))^2 +((√2))^2 ))  (1/( (√2)))tan^(−1) (((x−(1/x))/( (√2)))) +C
$$\int\frac{\frac{{x}^{\mathrm{2}} +\mathrm{1}}{{x}^{\mathrm{2}} }}{\frac{{x}^{\mathrm{4}} +\mathrm{1}}{{x}^{\mathrm{2}} }}{dx} \\ $$$$\int\frac{\mathrm{1}+\frac{\mathrm{1}}{{x}^{\mathrm{2}} }}{{x}^{\mathrm{2}} +\frac{\mathrm{1}}{{x}^{\mathrm{2}} }}{dx} \\ $$$$\int\frac{{d}\left({x}−\frac{\mathrm{1}}{{x}}\right)}{\left({x}−\frac{\mathrm{1}}{{x}}\right)^{\mathrm{2}} +\left(\sqrt{\mathrm{2}}\right)^{\mathrm{2}} } \\ $$$$\frac{\mathrm{1}}{\:\sqrt{\mathrm{2}}}\mathrm{tan}^{−\mathrm{1}} \left(\frac{{x}−\frac{\mathrm{1}}{{x}}}{\:\sqrt{\mathrm{2}}}\right)\:+{C} \\ $$
Commented by john santu last updated on 29/Mar/20
good answer
$$\mathrm{good}\:\mathrm{answer} \\ $$
Commented by som(math1967) last updated on 29/Mar/20
Thank you sir
$${Thank}\:{you}\:{sir} \\ $$
Answered by MJS last updated on 29/Mar/20
other method:  ∫((x^2 +1)/(x^4 +1))dx=  =(1/2)∫(dx/(x^2 −(√2)x+1))+(1/2)∫(dx/(x^2 +(√2)x+1))=  =((√2)/2)(arctan ((√2)x−1) +arctan ((√2)x+1))+C
$$\mathrm{other}\:\mathrm{method}: \\ $$$$\int\frac{{x}^{\mathrm{2}} +\mathrm{1}}{{x}^{\mathrm{4}} +\mathrm{1}}{dx}= \\ $$$$=\frac{\mathrm{1}}{\mathrm{2}}\int\frac{{dx}}{{x}^{\mathrm{2}} −\sqrt{\mathrm{2}}{x}+\mathrm{1}}+\frac{\mathrm{1}}{\mathrm{2}}\int\frac{{dx}}{{x}^{\mathrm{2}} +\sqrt{\mathrm{2}}{x}+\mathrm{1}}= \\ $$$$=\frac{\sqrt{\mathrm{2}}}{\mathrm{2}}\left(\mathrm{arctan}\:\left(\sqrt{\mathrm{2}}{x}−\mathrm{1}\right)\:+\mathrm{arctan}\:\left(\sqrt{\mathrm{2}}{x}+\mathrm{1}\right)\right)+{C} \\ $$
Commented by john santu last updated on 29/Mar/20
ostrogradetski?
$$\mathrm{ostrogradetski}? \\ $$
Commented by MJS last updated on 29/Mar/20
no. just decomposing
$$\mathrm{no}.\:\mathrm{just}\:\mathrm{decomposing} \\ $$

Leave a Reply

Your email address will not be published. Required fields are marked *